Survey Adjustments And Theory Of Errors(Surveying ) Questions and Answers

Question 1. Most probable value determines certain observations. Which among them will give accurate value?
  1.    Direct observation with unequal weights
  2.    Indirect observation with weights
  3.    Direct observation with weights
  4.    Observation with condition equation
Explanation:-
Answer: Option D. -> Observation with condition equation
Answer: (d).Observation with condition equation

Question 2. Determine the most probable value of B from the equations B = 65˚21ꞌ40ꞌꞌ weighing 3 and 2B = 132˚40ꞌ20ꞌꞌ weighing 2.
  1.    141˚18ꞌ52.7ꞌꞌ
  2.    114˚18ꞌ52.7ꞌꞌ
  3.    114˚8ꞌ52.7ꞌꞌ
  4.    411˚18ꞌ52.7ꞌꞌ
Explanation:-
Answer: Option B. -> 114˚18ꞌ52.7ꞌꞌ
Answer: (b).114˚18ꞌ52.7ꞌꞌ

Question 3. From which of the following cases, the value of most probable value can be easily determined?
  1.    Indirect method
  2.    Direct method
  3.    Method of correlates
  4.    From observations
Explanation:-
Answer: Option C. -> Method of correlates
Answer: (c).Method of correlates

Question 4. From the set of observations (1.87+ 9.73+ 9.22), find the most probable value. The weights of the observations are given as 4, 7, 9 respectively.
  1.    2.97
  2.    9.72
  3.    7.92
  4.    7.29
Explanation:-
Answer: Option C. -> 7.92
Answer: (c).7.92

Question 5. Find the most probable value with the observations 2.76, 4.32, 9.87, 8.83 having equal weights.
  1.    6.45
  2.    6.54
  3.    4.65
  4.    5.46
Explanation:-
Answer: Option A. -> 6.45
Answer: (a).6.45

Question 6. The laws of weight are established on the basis of____________
  1.    Observed equation
  2.    Normal equation
  3.    Least squares
  4.    Probability equation
Explanation:-
Answer: Option C. -> Least squares
Answer: (c).Least squares

Question 7. In the laws of weight, weight is inversely proportional to length.
  1.    True
  2.    False
  3.    May be True or False
  4.    Can't say
Explanation:-
Answer: Option A. -> True
Answer: (a).True

Question 8. Weight of the equation remains unchanged even when the signs in the equation are changed.
  1.    True
  2.    False
  3.    May be True or False
  4.    Can't say
Explanation:-
Answer: Option A. -> True
Answer: (a).True

Question 9. Find the arithmetic mean if the angles and their weights were given as 20˚42ꞌ3ꞌꞌ, 20˚42ꞌ4ꞌꞌ, 20˚42ꞌ6ꞌꞌ and 2, 2, 2 respectively.
  1.    20˚42ꞌ6.3ꞌꞌ
  2.    20˚42ꞌ5.3ꞌꞌ
  3.    20˚42ꞌ1.3ꞌꞌ
  4.    20˚42ꞌ4.3ꞌꞌ
Explanation:-
Answer: Option D. -> 20˚42ꞌ4.3ꞌꞌ
Answer: (d).20˚42ꞌ4.3ꞌꞌ

Question 10. Determine the weight of the weighted arithmetic mean if the angles and their weights are given as 40˚56ꞌ2ꞌꞌ, 40˚56ꞌ7ꞌꞌ, 40˚56ꞌ12ꞌꞌ and 5, 4, 9 respectively.
  1.    13
  2.    18
  3.    81
  4.    10
Explanation:-
Answer: Option B. -> 18
Answer: (b).18